Complex analysis integral

In summary, the given integral can be evaluated by completing in the lower or upper x-plane, using a rectangle, and showing that the integrals along the vertical and upper-horizontal segments tend to 0 as R goes to infinity. Alternatively, the integral can be rewritten as an integral of e^(ikx) and completed in the upper x-plane.
  • #1
Physgeek64
247
11

Homework Statement


calculate ## \int_{-\infty}^{\infty}{\frac{2}{1+x^2} e^{-ikx}dx}##, where k is any positive number

Homework Equations

The Attempt at a Solution


So first consider the closed contour ##I= \int{\frac{2}{1+z^2} e^{-ikz}dx}##

We can choose the contour to be along the real axis ## [-R,R]## then along the semi circle for which ##z=Re^{i \theta}, [0, \pi] ##

then ##I= \int_{-\infty}^{\infty}{\frac{2}{1+x^2} e^{-ikx}dx} +\int_{0}^{\pi} {\frac{2}{1+(Re^{i \theta})^2} e^{-ikRe^{i\theta}}dx}##

I calculated the residue to be ## \pi e^{k} ##, with the poles being at ## z=i## and ##z=-i##

I think the integral around the semi-circle is supposed to go to zero in the limit of R going to infinity, but I am struggling to see why. I can't use Jordan's lemma (?) since it is not in the usual formal, and if i expand

##e^{-ikRe^{i\theta}} = e^{-ikR(cos(\theta)+isin(\theta))}##
##= e^{-ikRcos(\theta)}e^{kRsin(\theta)}##

but since ##\theta ## is between ##[0, \pi], sin(\theta)## is greater than, or equal to zero, meaning that the integrand does not drop off as R goes to infinity..

Any help would be greatly appreciated- thanks in advance :)
 
Last edited:
Physics news on Phys.org
  • #2
You should add dx to your integrals.
 
  • #3
Math_QED said:
You should add dx to your integrals.
Noted and corrected. Thank you :)
 
  • Like
Likes member 587159
  • #4
Physgeek64 said:

Homework Statement


calculate ## \int_{-\infty}^{\infty}{\frac{2}{1+x^2} e^{-ikx}dx}##, where k is any positive number

Homework Equations

The Attempt at a Solution


So first consider the closed contour ##I= \int{\frac{2}{1+z^2} e^{-ikz}dx}##

We can choose the contour to be along the real axis ## [-R,R]## then along the semi circle for which ##z=Re^{i \theta}, [0, \pi] ##

then ##I= \int_{-\infty}^{\infty}{\frac{2}{1+x^2} e^{-ikx}dx} +\int_{0}^{\pi} {\frac{2}{1+(Re^{i \theta})^2} e^{-ikRe^{i\theta}}dx}##

I calculated the residue to be ## \pi e^{k} ##, with the poles being at ## z=i## and ##z=-i##

I think the integral around the semi-circle is supposed to go to zero in the limit of R going to infinity, but I am struggling to see why. I can't use Jordan's lemma (?) since it is not in the usual formal, and if i expand

##e^{-ikRe^{i\theta}} = e^{-ikR(cos(\theta)+isin(\theta))}##
##= e^{-ikRcos(\theta)}e^{kRsin(\theta)}##

but since ##\theta ## is between ##[0, \pi], sin(\theta)## is greater than, or equal to zero, meaning that the integrand does not drop off as R goes to infinity..

Any help would be greatly appreciated- thanks in advance :)

For ##k > 0## completing in the upper x-plane won't work, because for ##x = t + i y## we have ##e^{-ikx}= e^{-ikt}\, e^{ky}##. which behaves badly for large ##y > 0##. However, we can complete in the lower x-plane instead.

Alternatively, we can see that for real ##x## the integrand has a real part that is an even function of ##x## and an imaginary part that is odd (so will integrate to 0); that is, we could obtain the same result if we replace ##e^{-ikx}## by ##\cos(kx)##. But then, we might as well extend this to ##e^{ikx}## without changing its value.

So, let's look at the integral with integrand ##f(x) = e^{ikx}/(1+x^2)##. Now we can complete in the upper x-plane, because##e^{ik(t+iy)} = e^{ikt} e^{-ky}##. However, completing along a semi-circle is not very convenient; using a rectangle is much easier. So, let's take the original integral from ##x = -R## to ##x = +R##, taking ##R \to \infty## eventually. Complete in a rectangle as follows: append the integral from ##R+i0## to ##R + iR## along the vertical, then from ##R + iR## to ##-R + iR## along an upper horizontal, then from ##-R + iR## down to ##-R + i0## along the other vertical. It is pretty easy to get bounds on each of the vertical and upper-horizontal integrals, and to show that they ##\to 0## as ##R \to \infty##.
 
Last edited:
  • #5
Ray Vickson said:
For ##k > 0## completing in the upper x-plane won't work, because for ##x = t + i y## we have ##e^{-ikx}= e^{-ikt}\, e^{ky}##. which behaves badly for large ##y > 0##. However, we can complete in the lower x-plane instead.

Alternatively, we can see that for real ##x## the integrand has a real part that is an even function of ##x## and an imaginary part that is odd (so will integrate to 0); that is, we could obtain the same result if we replace ##e^{-ikx}## by ##\cos(kx)##. But then, we might as well extend this to ##e^{ikx}## without changing its value.

So, let's look at the integral with integrand ##f(x) = e^{ikx}/(1+x^2)##. Now we can complete in the upper x-plane. However, completing along a semi-circle is not very convenient; using a rectangle is much easier. So, let's take the original integral from ##x = -R## to ##x = +R##, taking ##R \to \infty## eventually. Complete in a rectangle as follows: append the integral from ##R+i0## to ##R + iR## along the vertical, then from ##R + iR## to ##-R + iR## along an upper horizontal, then from ##-R + iR## down to ##-R + i0## along the other vertical. It is pretty easy to get bounds on each of the vertical and upper-horizontal integrals, and to show that they ##\to 0## as ##R \to \infty##.

Closing in the lower half plane produces the same problem. Though the exponential power will be negative, over the limit ##[ 0, -\pi], sin(\theta)## is negative, meaning the resultant power is positive, and the same problem occurs. (This may also be seen by making the substitution ##x \to (-x)## in my initial working).

Edit:
But x (more accurately z) itself is complex, not real, over either the semi-circle or rectangle so taking the real part of the function will not simply be a matter of changing the exponential into its ##cosx+isinx## form. For example the integrand is actually ##\frac{1}{1+R^2e^{2i\theta}} e^{-ikRe^{i\theta}}Rie^{i\theta} ## taking the real part of this is not as straightforward, surely ?

Sorry if I'm missing something obvious- thanks for the help
 
Last edited:
  • #6
Physgeek64 said:
Closing in the lower half plane produces the same problem. Though the exponential power will be negative, over the limit ##[ 0, -\pi], sin(\theta)## is negative, meaning the resultant power is positive, and the same problem occurs. (This may also be seen by making the substitution ##x \to (-x)## in my initial working).

Edit:
But x (more accurately z) itself is complex, not real, over either the semi-circle or rectangle so taking the real part of the function will not simply be a matter of changing the exponential into its ##cosx+isinx## form. For example the integrand is actually ##\frac{1}{1+R^2e^{2i\theta}} e^{-ikRe^{i\theta}}Rie^{i\theta} ## taking the real part of this is not as straightforward, surely ?

Sorry if I'm missing something obvious- thanks for the help

No: we start with the integral over the real line, and show that we can replace ##e^{-ikx}## by ##e^{ikx}## in that integral. Then we extend ##x## into the upper complex plane. Alternatively, complete in the lower x-plane, and use arguments similar to the one below.

Taking ##e^{ikx} = e^{ik(t + iy)} = e^{ikt} e^{-ky}## for ##t = R## and ##0 \leq y \leq R##, the right-hand vertical integral becomes
$$ I_1 = \int_0^R \frac{e^{ikR} e^{-ky}}{(R + iy)^2 + 1} i \, dy,$$.
Use the facts that ##|\int f| \leq \int |f|##, ##|a/b| = |a|/|b|## and ##|e^{ikt} e^{-ky}| = |e^{ikt}| e^{-ky} = e^{-ky}##. The magnitude of the denominator in ##|f|## is
$$|(R+iy)^2 + 1| = \sqrt{R^4+2R^2 y^2+y^4+2 R^2-2y^2+1} \geq \sqrt{R^4 + 2 R^2},$$
since ##2R^2 y^2 \geq 0## and ##y^4 - 2 y^2 \geq -1## for ##y \geq 0##. We thus have
$$|I_1| \leq \int_0^R \frac{e^{-ky}}{ \sqrt{R^4+2R^2}} dy \to 0 $$
as ##R \to \infty##.

Similar types of arguments apply to the other vertical integral and to the upper horizontal integral.

Trying something like the above on a semi-circular arc of radius ##R## would be much harder, so that is why I suggested not doing it that way.
 
Last edited:

1. What is complex analysis integral?

Complex analysis integral is a branch of mathematics that deals with the integration of functions of complex variables. It involves the study of complex-valued functions, also known as holomorphic functions, and their properties.

2. What are some applications of complex analysis integral?

Complex analysis integral is used in various fields such as physics, engineering, and economics. Some practical applications include solving differential equations, calculating electric and magnetic fields, and analyzing the behavior of fluid flow.

3. How is complex analysis integral different from real analysis integral?

Complex analysis integral is based on the concept of complex numbers, while real analysis integral deals with real numbers. Complex analysis also has a wider scope, as it allows for the study of more complex functions and their behavior.

4. What are some techniques used in complex analysis integral?

Some common techniques used in complex analysis integral include Cauchy's integral theorem and formula, Laurent series, and residue theorem. These techniques help in evaluating complex integrals and solving various problems in the field.

5. Why is complex analysis integral important?

Complex analysis integral is an essential tool in understanding and solving problems in various fields of mathematics and science. It provides a deeper understanding of functions and their behavior, and allows for the development of complex mathematical models to solve real-world problems.

Similar threads

  • Calculus and Beyond Homework Help
Replies
4
Views
109
  • Calculus and Beyond Homework Help
Replies
9
Views
2K
  • Calculus and Beyond Homework Help
Replies
5
Views
670
  • Calculus and Beyond Homework Help
Replies
3
Views
547
  • Calculus and Beyond Homework Help
Replies
6
Views
1K
  • Calculus and Beyond Homework Help
Replies
9
Views
142
  • Calculus and Beyond Homework Help
Replies
7
Views
927
  • Calculus and Beyond Homework Help
Replies
1
Views
929
  • Calculus and Beyond Homework Help
Replies
3
Views
248
  • Calculus and Beyond Homework Help
Replies
3
Views
784
Back
Top